[obm-l] Re: [obm-l] Re: [obm-l] Problema de álgebra

2017-09-15 Por tôpico Leonardo Joau
On Fri, 15 Sep 2017 at 18:42 Ralph Teixeira wrote: > Bom, suponho que queremos alguma solucao que nao use tecnicas de Calculo? > > Que tal assim: x, y e z sao raizes do polinomio: > > t^3-t^2+at-P=0 > > onde P eh o que voce quer maximizar. > > O polinomio f(t)=t^3-t^2+at-P

[obm-l] Re: [obm-l] Problema de álgebra

2017-09-15 Por tôpico Ralph Teixeira
Bom, suponho que queremos alguma solucao que nao use tecnicas de Calculo? Que tal assim: x, y e z sao raizes do polinomio: t^3-t^2+at-P=0 onde P eh o que voce quer maximizar. O polinomio f(t)=t^3-t^2+at-P sempre tem pelo menos uma raiz real (grau 3). Quando voce muda P, voce translada o

[obm-l] Problema de álgebra

2017-09-15 Por tôpico Leonardo Joau
Dados os reais x, y,z, tais que: x+y+z = 1 xy+xz+yz = a 0

[obm-l] Re: [obm-l] Re: [obm-l] Re: [obm-l] Problema difícil.

2017-09-13 Por tôpico Ralph Teixeira
Oi, Douglas. Acho que o que você fez é um bom começo. Vamos adaptar: pense ao invés nos números de 1009 a 2017 (conjunto A). i) Eles podem todos parear com os números de 1 a 1008? ii) Então pelo menos um produto usando os elementos de A vai dar NO MÍNIMO NO MÍNIMO... iii) Esse número do item

[obm-l] Re: [obm-l] Re: [obm-l] Problema difícil.

2017-09-13 Por tôpico Douglas Oliveira de Lima
Então Bernardo, eu pensei numa parada mas não tenho certeza , pensei que os números 997,998,999,...,1994 Não poderiam ocupar as posições de 1 a 1997, logo pelo menos um deles ocuparia uma posição não inferior a 998, aí pensei no 997.998=995006. Em 12 de set de 2017 18:39, "Bernardo Freitas Paulo

[obm-l] Re: [obm-l] Re: [obm-l] Re: [obm-l] Problema difícil.

2017-09-12 Por tôpico Israel Meireles Chrisostomo
Exatamente, aplique a desigualdade do rearranjo Em 12 de setembro de 2017 19:08, Leonardo Joau escreveu: > > On Tue, 12 Sep 2017 at 18:39 Bernardo Freitas Paulo da Costa < > bernardo...@gmail.com> wrote: > >> 2017-09-12 17:51 GMT-03:00 Douglas Oliveira de Lima >>

[obm-l] Re: [obm-l] Re: [obm-l] Problema difícil.

2017-09-12 Por tôpico Leonardo Joau
On Tue, 12 Sep 2017 at 18:39 Bernardo Freitas Paulo da Costa < bernardo...@gmail.com> wrote: > 2017-09-12 17:51 GMT-03:00 Douglas Oliveira de Lima > : > > Considere a sequência de números 1,2,3,4,5,...,2017. > > E uma certa ordenação deles a1, a2, a3, ..., a2017. >

[obm-l] Re: [obm-l] Problema difícil.

2017-09-12 Por tôpico Bernardo Freitas Paulo da Costa
2017-09-12 17:51 GMT-03:00 Douglas Oliveira de Lima : > Considere a sequência de números 1,2,3,4,5,...,2017. > E uma certa ordenação deles a1, a2, a3, ..., a2017. > Agora multiplique respectivamente os números das duas sequencias > determinando assim uma nova

[obm-l] Problema difícil.

2017-09-12 Por tôpico Douglas Oliveira de Lima
Considere a sequência de números 1,2,3,4,5,...,2017. E uma certa ordenação deles a1, a2, a3, ..., a2017. Agora multiplique respectivamente os números das duas sequencias determinando assim uma nova sequência 1.a1, 2.a2, 3.a3, ..., 2017.a2017. Qual o menor valor que o maior produto da última

Re: [obm-l] Problema estranho

2017-09-07 Por tôpico Anderson Torres
Bora lá... Pelo que a galera já demonstrou, o resultado vale se todos os números da sequência forem racionais. Agora, falta cobrir os irracionais. Considere - real eps>0 - inteiro m>0 - inteiros p_1, p_2, ... p_(2n+1) tais que, para todo i, vale |p_i-mx_i| < eps. A ideia é que se eps for bem

Re: [obm-l] Problema de grafos

2017-09-03 Por tôpico Daniel da Silva
Obrigado pela ajuda Esdras e Matheus. Daniel Rocha da Silva > Em 2 de set de 2017, às 13:23, Esdras Muniz > escreveu: > > Cada vértice pode ter como grau um número de 0 a n-1, porém o 0 e o n-1 > não podem ambos ser graus de vértices, pois se um tem grau n-1

Re: [obm-l] Problema de grafos

2017-09-02 Por tôpico Esdras Muniz
Cada vértice pode ter como grau um número de 0 a n-1, porém o 0 e o n-1 não podem ambos ser graus de vértices, pois se um tem grau n-1 então ele está ligado a todos os outros vértices. Então há apenas n-1 possibilidades para o grau de cada vértice. Pelo pcp há dois vértices com o mesmo grau. Em 2

Re: [obm-l] Problema de grafos

2017-09-02 Por tôpico Matheus Secco
Olá Daniel, veja que os graus podem variar de 0 até n - 1. Entretanto, não é possível ter um vértice com grau 0 e outro com grau n - 1. Desta forma, em vez de n possibilidades para o grau de cada vértice, há n - 1 possibilidades para o grau de cada vértice. Como há n vértices, pelo Princípio da

[obm-l] Problema de grafos

2017-09-02 Por tôpico Daniel Rocha
Bom dia, Seja G um grafo com n vértices, n maior que 1. Suponha que G não possua loops nem mais de uma aresta unindo pares de vértices. Prove que G possui dois vértices de graus iguais. Obrigado, Daniel -- Esta mensagem foi verificada pelo sistema de antiv�rus e acredita-se estar livre de

Re: [obm-l] Problema de Probabilidade

2017-08-08 Por tôpico Ralph Teixeira
Ah, se voce preferir, pode dividir a tabela por jogador mesmo, assim: /// A B CD E FG Total JV 60 60 60 60 45 45 25 355 JP 40 40 40 40 55 55 75 345 Tot 100 100 100 100 100 100 100 700 a) Pr(JV)=355/700 b) Pr(E|JV)=45/355 Abraco, Ralph.

[obm-l] Problema de Probabilidade

2017-08-08 Por tôpico Luiz Antonio Rodrigues
Olá, pessoal! Bom dia! Será que alguém pode me ajudar com o problema abaixo? Estou quebrando a cabeça e não consigo resolvê-lo. Muito obrigado e um abraço! Luiz Um jogador J entra em um torneio de tênis com jogos eliminatórios. Seu primeiro adversário será selecionado aleatoriamente a partir de

[obm-l] Re: [obm-l] Problema de função elementar

2017-07-17 Por tôpico Alexandre Antunes
Boa noite, Encontrei um resultado aproximado F (21/2017)=F(0,01)=0,054719 Não sei se fiz "besteiras", mas usando a expressão em b F (x/3) = F(x)/2 x=1 =》F(1/3)=F(1)/2=1/2 x=1/3 =》F(1/9)=F(1/3)/2=1/4 Generalizando x=1/3^n =》F(1/3^n)=1/2^n Por outro lado Para

[obm-l] Re: [obm-l] Re: [obm-l] Re: [obm-l] Problema de função elementar

2017-07-17 Por tôpico Guilherme Oliveira
Aproveitando o problema, quanto seria f (0,1)? Tenham uma boa noite, Guilherme Em 17/07/2017 12:45, "Pedro José" escreveu: Bom dia! Seguindo a linha proposta pelo Anderson. 7/3^6 < 21/2017 < 8/3^6 ==> F(21/2017)= F(7/3^6)=F(8/3^6) F(7/9) = 3/4. F(7/3^6) = F(7/9/3^4)=

[obm-l] Re: [obm-l] Re: [obm-l] Problema de função elementar

2017-07-17 Por tôpico Pedro José
Bom dia! Seguindo a linha proposta pelo Anderson. 7/3^6 < 21/2017 < 8/3^6 ==> F(21/2017)= F(7/3^6)=F(8/3^6) F(7/9) = 3/4. F(7/3^6) = F(7/9/3^4)= F(7/9)/2^4= 3/2^6= 3/64. Sds, PJMS Em 17 de julho de 2017 10:48, Pedro José escreveu: > Bom dia! > > Há uma restrição para

[obm-l] Re: [obm-l] Re: [obm-l] Problema de função elementar

2017-07-17 Por tôpico Pedro José
Bom dia! Há uma restrição para a função ser crescente. Portanto F(1) é máximo e F(1) = 1, logo não pode ser 87. tem que ser um valor menor ou igual a 1 e maior ou igual a zero. Sds, PJMS Em 15 de julho de 2017 20:54, Matheus Herculano < matheusherculan...@gmail.com> escreveu: > O resultado é

[obm-l] Re: [obm-l] Problema de função elementar

2017-07-15 Por tôpico Matheus Herculano
O resultado é 87 Em 13 de jul de 2017 09:51, "Douglas Oliveira de Lima" < profdouglaso.del...@gmail.com> escreveu: > Seja F uma função crescente definida para todo número real x, 0<=x<=1, tal > que > > a) F(0)=0 > > b) F(x/3)=F(x)/2 > > c) F(1-x)=1-F(x) > > Encontrar F(21/2017). >

[obm-l] Re: [obm-l] Problema de função elementar

2017-07-15 Por tôpico Anderson Torres
F(1) = 1 F(1/3)=1/2, F(2/3)=1/2 - logo, F(x) = 1/2 se x está em [1/3,2/3] F(1/9)=1/4, F(2/9)=1/4, F(7/9)=3/4, F(8/9)=3/4 logo, F(x) = 1/4 se x está em [1/9,2/9] e F(x) = 3/4 se x está em [7/9,8/9] Acho que a ideia é por aí: ver em que terço-médio cairá o valor 21/2017. Em 13 de julho de 2017

Re: [obm-l] Problema estranho

2017-07-15 Por tôpico Anderson Torres
Uma ideia pode ser tentar aproximar os reais para racionais e usar o argumento das potências, não? Em 11 de julho de 2017 18:21, Matheus Secco escreveu: > Oi Ralph, tava sem tempo de escrever, mas vou aproveitar a deixa porque você > já fez quase tudo. Acho que dá pra

[obm-l] Problema de função elementar

2017-07-13 Por tôpico Douglas Oliveira de Lima
Seja F uma função crescente definida para todo número real x, 0<=x<=1, tal que a) F(0)=0 b) F(x/3)=F(x)/2 c) F(1-x)=1-F(x) Encontrar F(21/2017). Douglas Oliveira -- Esta mensagem foi verificada pelo sistema de antiv�rus e acredita-se estar livre de perigo.

Re: [obm-l] Problema estranho

2017-07-11 Por tôpico Matheus Secco
Oi Ralph, tava sem tempo de escrever, mas vou aproveitar a deixa porque você já fez quase tudo. Acho que dá pra fazer o caso geral usando que os reais admitem uma base considerando como um espaço vetorial sobre os racionais. Em ter, 11 de jul de 2017 às 18:18, Ralph Teixeira

Re: [obm-l] Problema estranho

2017-07-11 Por tôpico Ralph Teixeira
Ah, melhor ainda: depois que seus números forem inteiros, some uma certa constante a todos eles de forma que um deles seja 0. Agora divida por 2, quantas vezes você quiser (eles vão ser sempre todos pares pelo argumento de paridade anterior!). Então são todos inteiros divisíveis por poências

Re: [obm-l] Problema estranho

2017-07-11 Por tôpico Ralph Teixeira
Bom, eu sei resolver se todos os números forem racionais. Deve ter um jeito de usar isso para o caso geral... A propriedade desse conjunto não se altera se todos os elementos do conjunto forem multiplicados por um mesmo número, nem se a gente somar uma certa constante a todos eles. Assim, *SE*

Re: [obm-l] Problema estranho

2017-07-11 Por tôpico Nowras Ali
Uma prova por indução me parece o melhor caminho. O Bernardo já provou para o caso base, basta agora tentar provar para n+1, assumindo verdadeiro para n. Tentarei resolver o problema assim que puder. Abraços, Nowras. Em 9 de julho de 2017 18:54, Otávio Araújo

Re: [obm-l] Problema estranho

2017-07-09 Por tôpico Otávio Araújo
Já tentei isso, porém não parece ajudar em muita coisa mas de qualquer forma obrigado > Em 9 de jul de 2017, às 18:00, Bernardo Freitas Paulo da Costa > escreveu: > > Não pensei muito, mas acho que você deveria tentar provar os casos n=1 > e n=2 "no braço" para

Re: [obm-l] Problema estranho

2017-07-09 Por tôpico Bernardo Freitas Paulo da Costa
Não pensei muito, mas acho que você deveria tentar provar os casos n=1 e n=2 "no braço" para ter a intuição. E, na verdade, o enunciado deveria ser: dados a_1, a_2, ... a_{2n+1} números reais, não necessariamente distintos, tais que, para cada escolha de 2n dentre eles, é possível separar em dois

Re: [obm-l] Problema estranho

2017-07-09 Por tôpico Luiz Antonio Rodrigues
Olá, Francisco! Eu também pensei nisso, mas vou consultar o site que o Bruno indicou... Muito obrigado e um abraço! Luiz On Jul 8, 2017 9:13 PM, "Francisco Barreto" wrote: > > On Sat, 8 Jul 2017 at 20:21 Otávio Araújo > wrote: > >> >> O

Re: [obm-l] Problema estranho

2017-07-09 Por tôpico Luiz Antonio Rodrigues
Olá, Bruno! Muito obrigado pelo esclarecimento! Um abraço! Luiz On Jul 8, 2017 8:01 PM, "Bruno Visnadi" wrote: > Tecnicamente não dá para chamar de conjunto, quando há números repetidos. > O correto seria Multiconjunto: https://pt.wikipedia.org/wiki/Multiconjunto >

Re: [obm-l] Problema estranho

2017-07-08 Por tôpico Francisco Barreto
On Sat, 8 Jul 2017 at 20:21 Otávio Araújo wrote: > > O enunciado original eu não vi, quem me falou desse problema foi um amigo > meu. assim me perdoe pelo erro grosseiro. Mas considerando esse A um > multiconjunto, essa questão é verdadeira ou se tem um contra-exemplo?

Re: [obm-l] Problema estranho

2017-07-08 Por tôpico Francisco Barreto
On Sat, 8 Jul 2017 at 17:35 Otávio Araújo wrote: > Galera, queria que alguém pudesse resolver essa questão pra mim ( passei > muito tempo nela já kkk): > " Seja n um natural positivo e A um conjunto de 2n+1 números reais, não > necessariamente distintos, com a

Re: [obm-l] Problema estranho

2017-07-08 Por tôpico Otávio Araújo
O enunciado original eu não vi, quem me falou desse problema foi um amigo meu. assim me perdoe pelo erro grosseiro. Mas considerando esse A um multiconjunto, essa questão é verdadeira ou se tem um contra-exemplo? > Em 8 de jul de 2017, às 19:47, Bruno Visnadi >

Re: [obm-l] Problema estranho

2017-07-08 Por tôpico Bruno Visnadi
Tecnicamente não dá para chamar de conjunto, quando há números repetidos. O correto seria Multiconjunto: https://pt.wikipedia.org/wiki/Multiconjunto Em 8 de julho de 2017 19:27, Luiz Antonio Rodrigues escreveu: > Olá, Otávio! > Desculpe a intromissão. Eu não sei como

Re: [obm-l] Problema estranho

2017-07-08 Por tôpico Luiz Antonio Rodrigues
Olá, Otávio! Desculpe a intromissão. Eu não sei como resolver seu problema, mas quero aproveitá-lo para colocar uma questão que me atormenta desde a faculdade: pode existir um conjunto {1,1,1,2,3}? O número 1 não é único? Um abraço! Luiz On Jul 8, 2017 5:35 PM, "Otávio Araújo"

[obm-l] Problema estranho

2017-07-08 Por tôpico Otávio Araújo
Galera, queria que alguém pudesse resolver essa questão pra mim ( passei muito tempo nela já kkk): " Seja n um natural positivo e A um conjunto de 2n+1 números reais, não necessariamente distintos, com a seguinte propriedade: - Todo subconjunto de A com 2n elementos pode ser particionado em dois

Re: [obm-l] Problema da olimpiada hungara.

2017-05-23 Por tôpico Mauricio de Araujo
Obrigado!! -- Abraços, Mauricio de Araujo [oɾnɐɹɐ ǝp oıɔıɹnɐɯ] 2017-05-22 21:33 GMT-03:00 Pedro José : > Boa noite. > > Tentei da última vez escrever de uma forma simples, mas não deu, > tem muitas falhas, não vale, > > Na verdade, vai se formar um

Re: [obm-l] Problema da olimpiada hungara.

2017-05-22 Por tôpico Pedro José
Boa noite. Tentei da última vez escrever de uma forma simples, mas não deu, tem muitas falhas, não vale, Na verdade, vai se formar um período a partir da anomalia do algarismo das dezenas que é 1 e é a única vez que ele aparece. Depois será formado um período 023456789, que irá valer a

Re: [obm-l] Problema da olimpiada hungara.

2017-05-19 Por tôpico Mauricio de Araujo
A resposta é: 0. -- Abraços, Mauricio de Araujo [oɾnɐɹɐ ǝp oıɔıɹnɐɯ] 2017-05-19 12:18 GMT-03:00 Jackson Sousa : > Onde conferimos a resposta da questão? > > > Em 17 de maio de 2017 09:16, Bernardo Freitas Paulo da Costa < >

Re: [obm-l] Problema da olimpiada hungara.

2017-05-19 Por tôpico Jackson Sousa
Onde conferimos a resposta da questão? Em 17 de maio de 2017 09:16, Bernardo Freitas Paulo da Costa < bernardo...@gmail.com> escreveu: > É bem mais fácil. "Monte" o produto N*N como na escola. Vai ficar um > monte de "1" em cada linha e coluna. A 73ª coluna tem 73 "uns". > Agora, é só ver

Re: [obm-l] Problema da olimpiada hungara.

2017-05-17 Por tôpico Pedro José
Bom dia! Minha dúvida é de interpretação do português e não quanto a matemática. Quando se fala septuagésima terceira posição a partir do algarismo das unidades, fica dúvida inclusive ou exclusive? É mais fácil perguntar o algarismo de ordem 10^a. pois, dessa forma ficaria claro. Vou supor que é

Re: [obm-l] Problema da olimpiada hungara.

2017-05-17 Por tôpico Bernardo Freitas Paulo da Costa
É bem mais fácil. "Monte" o produto N*N como na escola. Vai ficar um monte de "1" em cada linha e coluna. A 73ª coluna tem 73 "uns". Agora, é só ver qual foi o "vai-um" da coluna anterior. E para isso tem que ver a anterior da anterior, mas (dica) não precisa ir muito longe. Abraços, --

Re: [obm-l] Problema da olimpiada hungara.

2017-05-16 Por tôpico Anderson Torres
N=99...9/9 = (10^2012-1)/9 9N = 10^2012-1 81N^2= 10^4024-2*10^2012+1 Agora tenta aplicar módulo 10^74: 81N^2= 10^4024-2*10^2012+1 81N^2=1 (mod 10^74) Agora teria que achar o "inverso" de 81 módulo 10^74, mas não parece fácil de cara. Outra forma seria usar alguma indução. Pelo que vi no

[obm-l] Problema da olimpiada hungara.

2017-05-16 Por tôpico Mauricio de Araujo
Dado o numero N = 1...11 formado por 2012 algarismos iguais a 1, qual o algarismo que ocupa a 73a. posição a partir do algarismo das unidades do numero N^2? -- Abraços, Mauricio de Araujo [oɾnɐɹɐ ǝp oıɔıɹnɐɯ] -- Esta mensagem foi verificada pelo sistema de antiv�rus e

[obm-l] Re: [obm-l] Problema Geometria

2017-05-09 Por tôpico Julio César Saldaña
Solução um pouco longa: - PB=PE - ABEC é inscritível => triângulo MEP = triângulo PEC (LAL). Por tanto

[obm-l] Problema Geometria

2017-05-09 Por tôpico vinicius raimundo
No triângulo ABC, AB=AC. D é um ponto sobre o lado BC tal que BD=2CD. Se P é o ponto de AD tal que

Re: [obm-l] Problema

2017-04-15 Por tôpico Luiz Antonio Rodrigues
Olá, Bruno! Muito obrigado! Gostei muito da sua solução. Uma ótima Páscoa para você! Um abraço! Luiz On Apr 15, 2017 1:57 PM, "Bruno Visnadi" wrote: > Bom, o que importa não é quantas vezes elas comem por dia, e sim o quanto > elas comem durante cada dia. Digamos

Re: [obm-l] Problema

2017-04-15 Por tôpico Bruno Visnadi
Bom, o que importa não é quantas vezes elas comem por dia, e sim o quanto elas comem durante cada dia. Digamos que todas as 16 vacas juntas comam N quilos de ração por dia, e temos 62N quilos ao total. Após 14 dias, sobram 48N quilos. Então ele vende 4 vacas, e a taxa de consumo passa a ser 3N/4

[obm-l] Problema

2017-04-15 Por tôpico Luiz Antonio Rodrigues
Olá, pessoal! Bom dia! Eu resolvi o problema abaixo supondo que as vacas comem uma vez por dia. Escrevi para perguntar se alguém consegue resolver de outra forma. A resposta é 28 dias. Muito obrigado, um abraço e uma ótima Páscoa para todos. Um fazendeiro possui ração suficiente para alimentar

Re: [obm-l] Problema de geometria.

2016-11-02 Por tôpico Bruno Visnadi
De acordo com o site http://objetoseducacionais2.mec.gov.br/bitstream/handle/mec/10396/geo0500.htm Flecha é um segmento de reta que une o ponto médio de uma corda ao ponto médio do arco correspondente. Em 2 de novembro de 2016 20:06, Tarsis Esau escreveu: > Se essas

Re: [obm-l] Problema de geometria.

2016-11-02 Por tôpico Tarsis Esau
Se essas "flechas" forem lados o triângulo não existe. Em 02/11/2016 6:16 PM, "Esdras Muniz" escreveu: > O que são essas "flechas"? > > Em 2 de novembro de 2016 17:57, Douglas Oliveira de Lima < > profdouglaso.del...@gmail.com> escreveu: > >> Olá amigos , preciso de

Re: [obm-l] Problema de geometria.

2016-11-02 Por tôpico Esdras Muniz
O que são essas "flechas"? Em 2 de novembro de 2016 17:57, Douglas Oliveira de Lima < profdouglaso.del...@gmail.com> escreveu: > Olá amigos , preciso de uma ajuda na seguinte questão, na verdade a > resolução porque já tentei muita coisa, já aprendi muita coisa com ela, mas > mesmo assim não a

[obm-l] Problema de geometria.

2016-11-02 Por tôpico Douglas Oliveira de Lima
Olá amigos , preciso de uma ajuda na seguinte questão, na verdade a resolução porque já tentei muita coisa, já aprendi muita coisa com ela, mas mesmo assim não a resolvi. As três flechas dos três lados (cordas) de um triângulo ABC inscrito em uma circunferência de raio R valem 1, 2 e 3 calcular a

Re: [obm-l] Problema

2016-10-08 Por tôpico Matheus Herculano
Para de me mandar isdo Em 8 de out de 2016 08:12, "regis barros" escreveu: > Bom dia > segue o problema > se x^y = 2 e y^x = 3, encontrar os valores de x e y. > > Grato > > Regis > > > Em Quarta-feira, 5 de Outubro de 2016 18:01, vinicius raimundo < >

Re: [obm-l] Problema

2016-10-08 Por tôpico Ralph Teixeira
Oi, Regis. Eu acho (acho!) que nao dah para resolver esse sistema no braco com as funcoes elementares usuais. Eliminando uma das variaveis, recai em algo do tipo: ln(lny)+(ln2)/y=ln(ln3) ou ln(lnx)+(ln3)/x=ln(ln2) E, ateh onde eu consigo pensar, equacoes desse tipo nao se resolvem no braco.

[obm-l] Problema

2016-10-08 Por tôpico regis barros
Bom diasegue o problemase x^y = 2 e y^x = 3, encontrar os valores de x e y. Grato Regis Em Quarta-feira, 5 de Outubro de 2016 18:01, vinicius raimundo escreveu: Obrigado Douglas  Em quarta-feira, 5 de outubro de 2016, Douglas Oliveira de Lima

[obm-l] Problema

2016-10-06 Por tôpico regis barros
Boa noite pessoalSe y^x=2 e x^y=3, encontrar os valores de x e y? Regis -- Esta mensagem foi verificada pelo sistema de antiv�rus e acredita-se estar livre de perigo.

Re: [obm-l] Problema

2016-07-11 Por tôpico Lucas Melo
Muito obrigado!! Enviado do meu iPhone > Em 8 de jul de 2016, às 13:47, Pedro José escreveu: > > Boa tarde! > > Primeiro, entendo que houve um erro no enunciado do problema, destacado em > amarelo. Deveria com raízes inteiras a1 e b1 e não a1 e a2 como escrito. > O

Re: [obm-l] Problema

2016-07-08 Por tôpico Pedro José
Boa tarde! Primeiro, entendo que houve um erro no enunciado do problema, destacado em amarelo. Deveria com raízes inteiras a1 e b1 e não a1 e a2 como escrito. O problema é meio controverso Pois não existe apenas uma equação. Qual o critério para a1 e b1? Se for assim: E dada uma equacao do

[obm-l] Problema

2016-07-08 Por tôpico Lucas Melo
Ola, alguem poderia me ajudar nesse problema ? E dada uma equacao do segundo grau x^2 + ax + b= 0 com raizes inteiras a1 e a2. Consideramos a equacao do segundo grau x^2 + a1x + b1=0. Se a equacao x^2 + a1x + b1=0 tem raizes inteiras a2 e b2, consideramos a equacao x^2 + a2x + b2 = 0. Se a

Re: [obm-l] Problema sobre trilha com subidas e descidas

2016-05-30 Por tôpico Pedro José
Boa tarde! Não sou professor. Sou leigo. Sou só um adimirador da matemática. Saudações, PJMS Em 28 de maio de 2016 13:38, Marcelo Gomes escreveu: > Olá professor Pedro, muito obrigado! > > Pois é, na minha cabeça, 3Km/h de velocidade, indicavam um trecho de 3 Km, >

Re: [obm-l] Problema sobre trilha com subidas e descidas

2016-05-28 Por tôpico Marcelo Gomes
Olá professor Pedro, muito obrigado! Pois é, na minha cabeça, 3Km/h de velocidade, indicavam um trecho de 3 Km, percorridos no tempo de 1 hora. Outra coisa que não havia compreendido é a questão da volta. Na minha cabeça, a trilha teria início no ponto A e fim em um ponto B. Obrigado pelas

Re: [obm-l] Problema sobre trilha com subidas e descidas

2016-05-27 Por tôpico Pedro José
Boa tarde! Seja *a* o trecho de subida e *b* o trecho de descida na ida para cahoeira teremos que *b *será o trecho de subida e *a* o trecho de descida na volta. Portanto: a/3 + b/4 = 3 2/3 a/4 + b/3 = 3 1/3 Resolvendo o sistema a = 8 km e b = 4km. Portanto o comprimento de cada perna é 12 km.

[obm-l] Problema sobre trilha com subidas e descidas

2016-05-27 Por tôpico Marcelo Gomes
Olá a todos, Estou resolvendo uma prova em um concurso para professor e n~~ao encontrei o gabarito desta que segue abaixo. Se alguém puder explicar, agradeço muito. O gabarito é 12, eu achei 23. "Para tomar um banho de cachoeira, Marcelo percorre uma trilha que não tem trechos planos, é

Re: [obm-l] Problema 6 da OBM de 2002

2015-10-12 Por tôpico Bernardo Freitas Paulo da Costa
2015-10-12 0:31 GMT-03:00 Gabriel Tostes : > Mostre que não podemos formar mais que 4096 sequências binárias de tamanho 24 > tal que quaisquer 2 diferem em ao menos 8 posições. > Não consegui entender a resolução na Eureka. Alguém pode resolvê-lo? Eu não sei se conheço alguma

RE: [obm-l] Problema 6 da OBM de 2002

2015-10-12 Por tôpico Esdras Muniz
Em qual EUREKA está a solução deste problema? -Mensagem Original- De: "Bernardo Freitas Paulo da Costa" <bernardo...@gmail.com> Enviada em: ‎12/‎10/‎2015 12:29 Para: "Lista de E-mails da OBM" <obm-l@mat.puc-rio.br> Assunto: Re: [obm-l] Problema 6 da OBM de

[obm-l] Problema 6 da OBM de 2002

2015-10-11 Por tôpico Gabriel Tostes
Mostre que não podemos formar mais que 4096 sequências binárias de tamanho 24 tal que quaisquer 2 diferem em ao menos 8 posições. Não consegui entender a resolução na Eureka. Alguém pode resolvê-lo? Sent from my iPad -- Esta mensagem foi verificada pelo sistema de antiv�rus e acredita-se

[obm-l] Re: [obm-l] Re: [obm-l] Re: [obm-l] Re: [obm-l] Problema muito bacana de teoria dos números

2015-08-08 Por tôpico Douglas Oliveira de Lima
Bom , vamos lá: 1)Como N possui 12 divisores, temos que 1 será o menor e N será o maior. 2)Usando uma propriedade bem conhecida teremos dk.d(13-k)=t, ou seja o divisor de indice k e o de índice 13-k. 3)Como o divisor de índice d4-1 é igual a (d1+d2+d4)d8, teremos que d1+d2+d4 é divisor também

[obm-l] Re: [obm-l] Re: [obm-l] Re: [obm-l] Problema muito bacana de teoria dos números

2015-08-07 Por tôpico Pedro José
Boa tarde! Saulo, Se 2 e 3 são divisores 6 também será. Achei esse problema casca grossa. Saudações, PJMS Em 6 de agosto de 2015 23:25, Mauricio de Araujo mauricio.de.ara...@gmail.com escreveu: N = 1989. Em 6 de agosto de 2015 14:50, saulo nilson saulo.nil...@gmail.com escreveu:

[obm-l] Re: [obm-l] Re: [obm-l] Problema muito bacana de teoria dos números

2015-08-06 Por tôpico Mauricio de Araujo
N = 1989. Em 6 de agosto de 2015 14:50, saulo nilson saulo.nil...@gmail.com escreveu: d4-1=11 d4=12 d1=1 d2=2 d3= d11=(1+2+12)d8=15*17=255 1,2,3,12,13,14,15,17,18,19,255, produto deles. 2015-08-06 13:14 GMT-03:00 Mauricio de Araujo mauricio.de.ara...@gmail.com: Um número natural N

[obm-l] Re: [obm-l] Problema muito bacana de teoria dos números

2015-08-06 Por tôpico saulo nilson
d4-1=11 d4=12 d1=1 d2=2 d3= d11=(1+2+12)d8=15*17=255 1,2,3,12,13,14,15,17,18,19,255, produto deles. 2015-08-06 13:14 GMT-03:00 Mauricio de Araujo mauricio.de.ara...@gmail.com : Um número natural N tem exatamente 12 divisores (incluindo 1 e N), tais que, colocados em ordem crescente temos d1

[obm-l] Re: [obm-l] Problema muito bacana de teoria dos números

2015-08-06 Por tôpico saulo nilson
d4-1=11 d4=12 d1=1 d2=2 d3= d11=(1+2+12)d8=15*17=255 1,2,3,12,13,14,15,17,18,19,255,256 2015-08-06 13:14 GMT-03:00 Mauricio de Araujo mauricio.de.ara...@gmail.com : Um número natural N tem exatamente 12 divisores (incluindo 1 e N), tais que, colocados em ordem crescente temos d1 d2 d3 ...

[obm-l] Problema muito bacana de teoria dos números

2015-08-06 Por tôpico Mauricio de Araujo
Um número natural N tem exatamente 12 divisores (incluindo 1 e N), tais que, colocados em ordem crescente temos d1 d2 d3 ... d12. Sabe-se que o divisor que possui o índice d4 - 1 é igual ao produto (d1 + d2 + d4).d8. Achar N. -- Abraços oɾnɐɹɐ ǝp oıɔıɹnɐɯ -- Esta mensagem foi verificada

[obm-l] Problema

2015-07-25 Por tôpico Israel Meireles Chrisostomo
Seja um triângulo inscrito numa circunferência de raio r, e seja os lados deste triângulo a,b,c.Seja uma esfera de raio 2r centrada nos pontos (x_0,y_0,z_0) .Seja um ponto qualquer no espaço tridimensional dado pelas coordenadas (x_i,y_ j,z_k) .Prove que dentre todas os valores das coordenadas

Re: [obm-l] Problema

2015-07-14 Por tôpico Benedito Tadeu V. Freire
Rogério, Olá. Muito obrigado. Benedito -- Open WebMail Project (http://openwebmail.org) -- Original Message --- From: Rogerio Ponce abrlw...@gmail.com To: obm-l@mat.puc-rio.br obm-l@mat.puc-rio.br Sent: Tue, 7 Jul 2015 19:43:31 -0300 Subject: Re: [obm-l] Problema Ola

RE: [obm-l] Problema

2015-07-09 Por tôpico benedito freire
Obrigado Gugu -Mensagem Original- De: g...@impa.br g...@impa.br Enviada em: ‎09/‎07/‎2015 17:08 Para: obm-l@mat.puc-rio.br obm-l@mat.puc-rio.br Cc: fe...@impa.br fe...@impa.br Assunto: Re: [obm-l] Problema Caro Benedito, Encaminho abaixo a solução do Renan Finder, que é ex

RE: [obm-l] Problema

2015-07-09 Por tôpico benedito freire
Obrigado Gugu -Mensagem Original- De: g...@impa.br g...@impa.br Enviada em: ‎09/‎07/‎2015 17:08 Para: obm-l@mat.puc-rio.br obm-l@mat.puc-rio.br Cc: fe...@impa.br fe...@impa.br Assunto: Re: [obm-l] Problema Caro Benedito, Encaminho abaixo a solução do Renan Finder, que é ex

Re: [obm-l] Problema

2015-07-09 Por tôpico gugu
Caro Benedito, Encaminho abaixo a solução do Renan Finder, que é ex-olímpico e aluno do IMPA, e mostra que A tem estratégia para ganhar: Chamamos de classe n o conjunto dos números remanescentes que são congruentes a n módulo 5. O jogador A vence se tentar minimizar a quantidade

Re: [obm-l] Problema

2015-07-07 Por tôpico Rogerio Ponce
? -- De: Mauricio de Araujo mauricio.de.ara...@gmail.com Enviada em: ‎01/‎07/‎2015 14:24 Para: obm-l@mat.puc-rio.br Assunto: Re: [obm-l] Problema ​ou melhor, A deve evitar enquanto puder apagar algum múltiplo de 5.​ Em 1 de julho de 2015 14:21, Mauricio de Araujo

RE: [obm-l] Problema

2015-07-06 Por tôpico benedito freire
Qual é realmente a estratégia para vencer? -Mensagem Original- De: Mauricio de Araujo mauricio.de.ara...@gmail.com Enviada em: ‎01/‎07/‎2015 14:24 Para: obm-l@mat.puc-rio.br obm-l@mat.puc-rio.br Assunto: Re: [obm-l] Problema ​ou melhor, A deve evitar enquanto puder apagar algum múltiplo

Re: [obm-l] Problema

2015-07-01 Por tôpico Mauricio de Araujo
petroc...@gmail.com Data: 1 de julho de 2015 10:54 Assunto: Re: [obm-l] Problema Para: obm-l@mat.puc-rio.br Bom dia! E={1,6,11,16,21,26} e F= {4,9,14,19,24} Para qualquer par (a,b) com a Ɛ E e b Ɛ F == a + b ≡ 0 (mod5). G= {2, 7, 12, 17, 22,27} e H = {3, 8, 13, 18, 23} Para qualquer (a,b

[obm-l] Problema

2015-07-01 Por tôpico benedito
Problema Dois jogadores, A e B, disputam um jogo, em que jogam alternadamente. O jogador A começa. Uma jogada consiste em apagar um dos números inteiros do conjunto {1, 2, 3,..., 27} até que reste somente dois números. Se a soma desses dois últimos números for divisível por 5, o jogador A

Re: [obm-l] Problema

2015-07-01 Por tôpico Pedro José
Bom dia! E={1,6,11,16,21,26} e F= {4,9,14,19,24} Para qualquer par (a,b) com a Ɛ E e b Ɛ F == a + b ≡ 0 (mod5). G= {2, 7, 12, 17, 22,27} e H = {3, 8, 13, 18, 23} Para qualquer (a,b) com a Ɛ G e b Ɛ H == a + b ≡ 0 (mod5). J= {5, 15, 20, 25} Para qualquer par (a,b) com a,b Ɛ J== a + b ≡ 0 (mod5).

Re: [obm-l] Problema

2015-07-01 Por tôpico Mauricio de Araujo
a sobra de E ou F antes de cabarem todos os números. Necessita de reanálise. -- Mensagem encaminhada -- De: Pedro José petroc...@gmail.com Data: 1 de julho de 2015 10:54 Assunto: Re: [obm-l] Problema Para: obm-l@mat.puc-rio.br Bom dia! E={1,6,11,16,21,26} e F= {4,9,14,19,24

Fwd: [obm-l] Problema

2015-07-01 Por tôpico Pedro José
Bom dia ! Está errado o jogador pode escolher a sobra de E ou F antes de cabarem todos os números. Necessita de reanálise. -- Mensagem encaminhada -- De: Pedro José petroc...@gmail.com Data: 1 de julho de 2015 10:54 Assunto: Re: [obm-l] Problema Para: obm-l@mat.puc-rio.br Bom dia

Re: [obm-l] Problema

2015-07-01 Por tôpico Mauricio de Araujo
petroc...@gmail.com Data: 1 de julho de 2015 10:54 Assunto: Re: [obm-l] Problema Para: obm-l@mat.puc-rio.br Bom dia! E={1,6,11,16,21,26} e F= {4,9,14,19,24} Para qualquer par (a,b) com a Ɛ E e b Ɛ F == a + b ≡ 0 (mod5). G= {2, 7, 12, 17, 22,27} e H = {3, 8, 13, 18, 23} Para qualquer (a,b

[obm-l] Re: [obm-l] Problema da 18ª Olimpíada de Maio

2015-06-26 Por tôpico Rogerio Ponce
Ola' Mariana, trace por M uma perpendicular ao lado BC, e chame de E sua intersecao com DB. Chame de F a intersecao de DM com CE. Por construcao, o triangulo EBC e' isosceles. Como CD e' perpendicular 'a CA, entao CD e' bissetriz ( externa ) do angulo entre o lado CD e o prolongamento do lado BC

Re: [obm-l] Problema

2015-06-26 Por tôpico Mauricio de Araujo
Existe uma solução para este problema na revista Eureka no. 5. Em 22 de junho de 2015 18:32, Douglas Oliveira de Lima profdouglaso.del...@gmail.com escreveu: Olá caros colegas, gostaria de uma ajuda no seguinte problema: Em uma reta há 1999 bolinhas. Algumas são verdes e as demais

[obm-l] Problema da 18ª Olimpíada de Maio

2015-06-22 Por tôpico Mariana Groff
Boa Tarde, No triângulo ABC, verificamos que B=2C e  o A 90 . Seja M o ponto médio de BC. A perpendicular por C ao lado AC corta a reta AB no ponto D. Demonstre que AMB=DMC. Obrigada, Mariana -- Esta mensagem foi verificada pelo sistema de antiv�rus e acredita-se estar livre de perigo.

[obm-l] Problema

2015-06-22 Por tôpico Douglas Oliveira de Lima
Olá caros colegas, gostaria de uma ajuda no seguinte problema: Em uma reta há 1999 bolinhas. Algumas são verdes e as demais azuis(poderiam ser todas verdes ou todas azuis). Debaixo de cada bolinha escrevemos o número igual a soma da quantidade de bolinhas verdes à sua direita dela mais a

Re: [obm-l] Problema de Desigualdade

2015-06-11 Por tôpico Douglas Oliveira de Lima
Então não é trabalhoso, mas (a/b)^2 = 1 + a/b - b/a não deveria ser provado? Desenvolvendo da pra ver que é, neste caso tem mais conta pra fazer. Forte abraço Douglas Oliveira. Em 10 de junho de 2015 12:00, Alexandre Antunes prof.alexandreantu...@gmail.com escreveu: Bom dia, Estou no

Re: [obm-l] Problema de Desigualdade

2015-06-11 Por tôpico Alexandre Antunes
Boa tarde, Pensei em fazer essa prova por indução ... Ainda não consegui parar para finalizar. Achei que era um caminho possível!!! Em 11/06/2015 14:28, Douglas Oliveira de Lima profdouglaso.del...@gmail.com escreveu: Então não é trabalhoso, mas (a/b)^2 = 1 + a/b - b/a não deveria ser

Re: [obm-l] Problema de Desigualdade

2015-06-10 Por tôpico Pacini Bores
Ok Mariana. Abraços Pacini Em 9 de junho de 2015 21:11, Mariana Groff bigolingroff.mari...@gmail.com escreveu: Oi Pacini, Fiz do seguinte modo: f (x)=x^2-x+1/x=1 = x^3-x^2+1=x = x^3-x^2-x+1=0 =x^2 (x-1)-(x-1)=0 = (x^2-1)(x-1)=0 O que podemos ver que é verdade, analisando ambos os casos:

Re: [obm-l] Problema de Desigualdade

2015-06-10 Por tôpico Alexandre Antunes
Bom dia, Estou no trabalho, mas vou arriscar a minha primeira resposta no grupo. Desenvolvi os dois lados da expressao. (a/b)^2 + (b/c)^2 + (c/a)^2 = 3 + (a/b + b/c + c/a) - (b/a + c/b + a/c) Como (a/b)^2 = 1 + a/b - b/a O mesmo para os demais termos Fica provado a proposição. O que acham

Re: [obm-l] Problema de Desigualdade

2015-06-09 Por tôpico Pacini Bores
Oi Mariana, Observe que provar a desigualdade pedida é equivalente provar que : {(a/b)^2-a/b+b/a} + {(b/c)^2-b/c+c/b} +{(c/a)^2-c/a+a/c} =3, ok ? Agora façamos o seguinte : Seja f(x)= x^2-x+1/x, verifique que para x0 o valor mínimo de f é 1. Donde teremos a desigualdade provada. Estou

RES: [obm-l] Problema Interessante de Geometria

2015-06-09 Por tôpico Albert Bouskela
Olá, Ralph, O arquivo GeoGebra (“Hexagons.ggb”) foi bloqueado pelo sistema que administra esta Lista, em face da possibilidade de vírus (por tratar-se de um arquivo executável). Peço, então, que envie o respectivo arquivo diretamente para o meu e-mail. Prometo (como sempre…) tentar

Re: [obm-l] Problema de Desigualdade

2015-06-09 Por tôpico Mariana Groff
Oi Pacini, Compreendi seu raciocínio. Para provar que f(x)=1, basta analisarmos que (x^2-1)(x-1)=0, o que verifica-se pois se x=1, o produto é claramente não-negativo e se 0x1, vemos que, tanto x^2-1 quanto x-1 são negativos, tornando o produto positivo, isso? Em 9 de junho de 2015 11:48,

Re: [obm-l] Problema de Desigualdade

2015-06-09 Por tôpico Pacini Bores
Oi Mariana, Determinei o mínimo da função usando a derivada. Não entendi o seu caminho, pois a função é f(x) = x^2-x+1/x. Abraços Pacini Em 9 de junho de 2015 18:09, Mariana Groff bigolingroff.mari...@gmail.com escreveu: Oi Pacini, Compreendi seu raciocínio. Para provar que f(x)=1,

<    1   2   3   4   5   6   7   8   9   10   >